Sum of series: using 1 + 1/2 + 1/2 +.... to show 1/n diverges

  • Thread starter Battlemage!
  • Start date
  • Tags
    Series Sum
In summary, the conversation discusses a problem in Boas 3rd edition that asks the reader to use a specific test to show that the harmonic series diverges. The individual attempted to rearrange the terms in the series to make smaller series, but this approach was not successful as it resulted in a convergent series instead of a divergent one. The individual's main mistake was not making a true rearrangement and leaving out many terms. This mistake is pointed out and the concept of Ramanujan summation is briefly mentioned as an alternative approach to handling divergent series.
  • #1
Battlemage!
294
45
<<Moderator's note: moved from a technical forum, so homework template missing.>>

I found a problem in Boas 3rd ed that asks the reader to use
[tex]S_n = 1 + \frac{1}{2} + \frac{1}{2} + \frac{1}{2} + ...[/tex]

to show that the harmonic series diverges. They specifically want this done using the test where
if |an| ≥ dn of a known divergent series, then the series for an also diverges.

In trying to do this, I rearranged the terms in Sn so that they made smaller series, all added together, with each one smaller than 1/n. In order for this to work in the way I'm trying to do it, the following has to be true:

[tex]\sum_{n=1}^\infty \frac{1}{2^n} = \sum_{n=1}^\infty \frac{1}{2^{n+1}} + \sum_{n=1}^\infty \frac{1}{2^{n+2}} + \sum_{n=1}^\infty \frac{1}{2^{n+3}} + ...[/tex]

Is that true? I know the limits as n approaches infinity all go to zero.

Here are the details of my attempt at this:

[tex]S_n = 1 + \frac{1}{2} + \frac{1}{2} + \frac{1}{2} + ...[/tex]
[tex]= 1 + \frac{1}{2} + (\frac{1}{4}+\frac{1}{4}) + (\frac{1}{8} + \frac{1}{8} +\frac{1}{8} +\frac{1}{8}) + (\frac{1}{16} + \frac{1}{16} +\frac{1}{16} +\frac{1}{16} + \frac{1}{16} + \frac{1}{16} +\frac{1}{16} +\frac{1}{16}) +...[/tex]

Then I rearranged them like this (since addition is commutative):

[tex]= 1 + \frac{1}{2} + \frac{1}{4}+\frac{1}{8} +...+ \frac{1}{4} + \frac{1}{8}+\frac{1}{16} +...+ \frac{1}{8} + \frac{1}{16}+\frac{1}{32}+ ... +[/tex]

and so on (the first 1 was giving me fits until I realized 1 = 1/2 + 1/2 anyway, so from this point on I ignored it).This all should then equal:

[tex]= \sum_{n=1}^\infty \frac{1}{2^n} + \sum_{n=1}^\infty \frac{1}{2^{n+1}} + \sum_{n=1}^\infty \frac{1}{2^{n+2}} + ...[/tex]

Where essentially it becomes a double sum with n going to infinity and then m going to infinity (i.e. the exponent is n, n+1, n+2, ... n+m). It's here that it occurred to me that n+m can just be k, and it's all a dummy variable anyway, which would mean that:

[tex]\sum_{n=1}^\infty \frac{1}{2^n} + \sum_{n=1}^\infty \frac{1}{2^{n+1}} + \sum_{n=1}^\infty \frac{1}{2^{n+2}} + ... = \sum_{n=1}^\infty \frac{1}{2^k}[/tex]

and since k=n is fine since it's a dummy variable, it just equals:

[tex]\sum_{n=1}^\infty \frac{1}{2^n}[/tex].

Since 1/2n< |1/n|, it then follows that 1/n diverges.
I appreciate you reading this. Please rip it apart. :)
 
Last edited by a moderator:
Physics news on Phys.org
  • #2
You did a lot of unnecessary work, and as result of that unnecessary work the proof does not work any more.

Battlemage! said:
[tex]\sum_{n=1}^\infty \frac{1}{2^n} = \sum_{n=1}^\infty \frac{1}{2^{n+1}} + \sum_{n=1}^\infty \frac{1}{2^{n+2}} + \sum_{n=1}^\infty \frac{1}{2^{n+3}} + ...[/tex]
This equation is true, but the left sum has a finite value. You want to compare the harmonic series to a divergent series, not a convergent one.
Battlemage! said:
Then I rearranged them like this (since addition is commutative):
A finite number of exchanges is okay, but rearrangements like you did do not work - they can change the limit of the series. In particular, with this step you made a convergent series out of your divergent one. Where is the point in everything that follows?

Battlemage! said:
[tex]S_n = 1 + \frac{1}{2} + \frac{1}{2} + \frac{1}{2} + ...[/tex]
[tex]= 1 + \frac{1}{2} + (\frac{1}{4}+\frac{1}{4}) + (\frac{1}{8} + \frac{1}{8} +\frac{1}{8} +\frac{1}{8}) + (\frac{1}{16} + \frac{1}{16} +\frac{1}{16} +\frac{1}{16} + \frac{1}{16} + \frac{1}{16} +\frac{1}{16} +\frac{1}{16}) +...[/tex]
This is the key point, and all you need in terms of series manipulation. The series is clearly divergent, and in the second line every term is smaller than the corresponding term of the harmonic series.
 
  • Like
Likes Battlemage!
  • #3
As mfb points out, the manipulations you've done break the rules of what an infinite sum means, so they are not valid deductions in classical mathematics. If you're interested in playing around with series though, and what happens if we relax some of the rules, you may be interested in Ramanujan summation, which is pretty topical given a movie just came out recently about Ramanujan.

The concept of a Ramanujan summation can be made rigorous, but it describes an operation that is different from what is accepted in mathematics as calculating the sum of a divergent infinite series. Not only can Ramanujan derive a finite 'sum' of a divergent series but it also can derive different 'sums' by different rearrangements of the order.
 
  • Like
Likes Battlemage!
  • #4
Since this a series with positive terms, its terms may be rearranged in an arbitrary manner, without changing the sum (which is considered to be ##+\infty## if the sum diverges.

In this case however, you begin with a divergent series and end up with a convergent one:

Battlemage! said:
[tex]S_n = 1 + \frac{1}{2} + \frac{1}{2} + \frac{1}{2} + ...[/tex]
[tex]= 1 + \frac{1}{2} + (\frac{1}{4}+\frac{1}{4}) + (\frac{1}{8} + \frac{1}{8} +\frac{1}{8} +\frac{1}{8}) + (\frac{1}{16} + \frac{1}{16} +\frac{1}{16} +\frac{1}{16} + \frac{1}{16} + \frac{1}{16} +\frac{1}{16} +\frac{1}{16}) +...[/tex]

Then I rearranged them like this (since addition is commutative):

[tex]= 1 + \frac{1}{2} + \frac{1}{4}+\frac{1}{8} +...+ \frac{1}{4} + \frac{1}{8}+\frac{1}{16} +...+ \frac{1}{8} + \frac{1}{16}+\frac{1}{32}+ ... +[/tex]

and so on
and the latter series in convergent, you find correctly its sum as equal to ##1+\sum_{n=1}^\infty\frac1{2^n}## which is a convergent geometric series, not a divergent series.

Your main mistake is that you did not make a true rearrangement, but left out many terms: in your first sum, you have one ##\frac12##-term, two ##\frac 14##-terms, four ##\frac18##-terms, eight ##\frac1{16}##-terms etc. while in the second (nested) series you have one ##\frac12##-tern, two ##\frac14##-terms, three ##\frac18##-terms, four ##\frac1{16}##-terms, etc.
So, you left out lots and lots of terms. Therefore, it is not sursprising that the first series diverges and the second converges.
 
  • #5
I don't see anywhere in this thread where the harmonic series is given. Look it up on Wikipedia or whatever.

Here it is: ##\ \displaystyle \sum _{n=1}^{\infty }\frac {1}{n}=1+\frac {1}{2}+\frac {1}{3}+\frac {1}{4}+{\frac {1}{5}}+\cdots ##
 
  • #6
Erland said:
Since this a series with positive terms, its terms may be rearranged in an arbitrary manner, without changing the sum (which is considered to be ##+\infty## if the sum diverges.

In this case however, you begin with a divergent series and end up with a convergent one:and the latter series in convergent, you find correctly its sum as equal to ##1+\sum_{n=1}^\infty\frac1{2^n}## which is a convergent geometric series, not a divergent series.

Your main mistake is that you did not make a true rearrangement, but left out many terms: in your first sum, you have one ##\frac12##-term, two ##\frac 14##-terms, four ##\frac18##-terms, eight ##\frac1{16}##-terms etc. while in the second (nested) series you have one ##\frac12##-tern, two ##\frac14##-terms, three ##\frac18##-terms, four ##\frac1{16}##-terms, etc.
So, you left out lots and lots of terms. Therefore, it is not sursprising that the first series diverges and the second converges.

Thanks! That definitely explains it in a very understandable way. My train of thought was that at some point along the road to infinity, each of those groupings should eventually add up to the preceding fraction. Of course that seems to be mistaken. But I can definitely see where I clearly missed terms as the series progressed.
SammyS said:
I don't see anywhere in this thread where the harmonic series is given. Look it up on Wikipedia or whatever.

Here it is: ##\ \displaystyle \sum _{n=1}^{\infty }\frac {1}{n}=1+\frac {1}{2}+\frac {1}{3}+\frac {1}{4}+{\frac {1}{5}}+\cdots ##
It was given (sloppily) in the last line of the OP:

Battlemage! said:
Since 1/2n< |1/n|, it then follows that 1/n diverges.
 

1. What is the sum of the series 1 + 1/2 + 1/2 + ...?

The sum of this series can be represented as S = 1 + 1/2 + 1/2 + ..., where the denominators increase by a factor of 2 each term. This is known as a geometric series with a common ratio of 1/2. The sum of a geometric series can be calculated using the formula S = a / (1 - r), where a is the first term and r is the common ratio. In this case, a = 1 and r = 1/2, so the sum is S = 1 / (1 - 1/2) = 2.

2. How does the sum of this series show that 1/n diverges?

The sum of the series 1 + 1/2 + 1/2 + ... can be represented as 1 + 1/2 + 1/4 + 1/8 + ..., where the denominators decrease by a factor of 2 each term. This is known as a geometric series with a common ratio of 1/2. As the number of terms in the series increases, the sum approaches the value of 2. However, the series 1/n does not have a finite sum, as the terms do not approach a specific value. Therefore, the series 1/n diverges.

3. Why is it important to determine if a series diverges?

Determining if a series diverges is important because it helps us understand the behavior of the series and its terms. If a series diverges, it means that the terms do not approach a finite value, which can have important implications in various fields of science, such as physics and economics. It also helps us determine the convergence of a series, which can be useful in solving mathematical problems.

4. Can the sum of a series be infinite?

Yes, the sum of a series can be infinite. A series is said to converge if the sum of its terms approaches a finite value as the number of terms increases. However, if the sum of the terms does not approach a finite value, the series is said to diverge and the sum is infinite. This is the case for the series 1/n, which has an infinite sum as the terms do not approach a specific value.

5. What is the difference between a series that diverges and one that converges?

A series that converges has a finite sum, meaning that the terms approach a specific value as the number of terms increases. On the other hand, a series that diverges does not have a finite sum, as the terms do not approach a specific value. This means that the sum of the terms in a divergent series can be infinite or undefined. In terms of mathematical calculations, a convergent series can be evaluated to a specific value, while a divergent series cannot be evaluated in the same way.

Similar threads

  • Calculus and Beyond Homework Help
Replies
3
Views
425
  • Calculus and Beyond Homework Help
Replies
1
Views
359
  • Calculus and Beyond Homework Help
Replies
1
Views
231
  • Calculus and Beyond Homework Help
Replies
1
Views
276
  • Calculus and Beyond Homework Help
Replies
2
Views
719
  • Calculus and Beyond Homework Help
Replies
17
Views
631
  • Calculus and Beyond Homework Help
Replies
17
Views
1K
  • Calculus and Beyond Homework Help
Replies
3
Views
500
  • Calculus and Beyond Homework Help
Replies
6
Views
485
Replies
8
Views
1K
Back
Top